A true conditional statement is below.
"If two angles form a linear pair, then the
two angles are supplementary."
Which of the following statements is false?
I.
If two angles are supplementary,
then they are a linear pair.
II.
If two angles are not a linear pair,
then they are not supplementary.
III. If two angles are not
supplementary, then they are not a
linear pair
A. I and III only
B. II and III only
C. I and II only
D. I, II, and III

Answers

Answer 1

Answer:

I and II only

Step-by-step explanation:


Related Questions

The following data points represent the volume of gas in each race car driver's tank (in liters). Sort the data from least to greatest. Find the interquartile range (IQR) of the data set. \text{L}Lstart text, L, end text \text{L}Lstart text, L, end text

Answers

Answer:

its \text{L}Lstart text, L, end text

Step-by-step explanation:

The sorting of the dataset and the information from the five number summary, which includes the three quartiles and the minimum and maximum values indicates that we get;

1. 2.8, 4, 4.3, 6, 7.5, 8.5, 9, 11.6, 12, 12.1

2. The interquartile range is; 7.745

What is the five number summary of interquartile range of a data set?

The five number summary includes the first, second and third quartiles, Q₁, Q₂, and Q₃, and maximum and minimum values of the dataset, which serves to describe the characteristics of the data.

here, we have,

1. The sorted data from least to greatest can be presented as follows;

2.8, 4, 4.3, 6, 7.5, 8.5, 9, 11.6, 12, 12.1

Therefore, the original data in the question is already sorted from least to the greatest

2. The interquartile range, IQR, is the the difference between the third quartile, Q₃, and the first quartiles, Q₁, of the data.

IQR = Q₃ - Q₁

The dataset indicates that we get;

The number of datapoints in the dataset, n = 10

The first quartile = The (N + 1)/4 percentile value

First quartile, Q₁ = (10 + 1)/4 × 10 = The 2.75th value

The 2.75th value is the value 0.75 = 3/4 larger than the 2nd value, which can be found as follows;

Q₁ = The 2.75th value = 4 + (4.3 - 4) × (3/4)  = 4.225

The third quartile, Q₃ = The (N + 1) × (3/40th value, therefore;

Q₃ = The (10 + 1) × (3/4)th value = The 8.25th value (The value 0.25th more than the 8th value )

Q₃ = 11.6 + (12 - 11.6) × 0.25 = 11.7

The interquartile range, IQR = 11.7 - 4.225 = 7.475

The interquartile range = 7.475

Learn more on the interquartile range of a dataset here: brainly.com/question/13119591

#SPJ7

Complete question:

The following data points represent the volume of gas in each race car driver's tank (in liters).

Answer 2 questions about the data set.

1. Sort the data from least to greatest.

2.8

4

4.3

6

7.5

8.5

9

11.6

12

12.1

2. Find the interquartile range (IQR) of the data set.

L

how do i solve this? xw+vy-xy-vw​

Answers

Here you go hopes this helps

Identify the error in each problem. Explain each error and provide the correction solution. Use at least two complete sentences. PLEASE SOMEONE HELP QUICK ITS DUE TOMORROW AND I WILL MARK YOU BRAINLIEST

Answers

Answer:

the mistake is the addition sign. When a number is next to a parenthesis it means multiplication

How much of the circle is shaded? simply your answer

Answers

Answer:

The part that is shaded is 5/14

Step-by-step explanation:

The total circle is 1

1/2 + 1/7 + x = 1  where x is the shaded area

Multiply each side by 14 to get rid of the fractions

14(1/2 + 1/7 + x) = 1*14

7 + 2 + 14x = 14

9+14x = 14

Subtract 9 from each side

14x = 5

Divide by 14

x = 5/14

The part that is shaded is 5/14

find 3+root 2/3-root 2=a+b root2

Answers

Answer:

a = [tex]\frac{11}{7}[/tex] ; b = [tex]\frac{6}{7}[/tex]

Step-by-step explanation:

[tex]\frac{3 + \sqrt{2}}{3 - \sqrt{2} } = a + b\sqrt{2} \\\\[/tex]

Rationalising [tex]\frac{3 + \sqrt{2}}{3 - \sqrt{2} }[/tex] gives :-

[tex]\frac{3 + \sqrt{2}}{3 - \sqrt{2} } = \frac{(3 + \sqrt{2})(3 + \sqrt{2})}{(3 - \sqrt{2})(3 + \sqrt{2}) } = \frac{(3 + \sqrt{2})^2}{3^2 - (\sqrt{2})^2 } = \frac{11 +6\sqrt{2} }{7}[/tex]

Comparing [tex]\frac{11 + 6\sqrt{2} }{7}[/tex] with [tex]a + b\sqrt{2}[/tex] gives

a = [tex]\frac{11}{7}[/tex] & b = [tex]\frac{6}{7}[/tex]

Answer:

[tex]\frac{11}{7}[/tex] + [tex]\frac{6}{7}[/tex] [tex]\sqrt{2}[/tex]

Step-by-step explanation:

Given

[tex]\frac{3+\sqrt{2} }{3-\sqrt{2} }[/tex]

Multiply the numerator/ denominator by the conjugate of the denominator.

The conjugate of 3 - [tex]\sqrt{2}[/tex] is 3 + [tex]\sqrt{2}[/tex] , thus

= [tex]\frac{(3+\sqrt{2})(3+\sqrt{2}) }{(3-\sqrt{2})(3+\sqrt{2}) }[/tex] ← expand numerator/ denominator using FOIL

= [tex]\frac{9+6\sqrt{2}+2 }{9-2}[/tex]

= [tex]\frac{11+6\sqrt{2} }{7}[/tex]

= [tex]\frac{11}{7}[/tex] + [tex]\frac{6}{7}[/tex] [tex]\sqrt{2}[/tex] ← in the form a + b[tex]\sqrt{2}[/tex]

with a = [tex]\frac{11}{7}[/tex] and b = [tex]\frac{6}{7}[/tex]

Use the table to complete the work to find the missing value. Conversion Chart Pints Ounces 3 48 7 ? 30 480 3 pints 48 ounces 11 pints ? ounces How many ounces are in 11 pints? o 144 o160 o 176 o 192​

Answers

Answer:

176 ounces (C)

Step-by-step explanation:

if you divide 48 by 3 it will give you 16 and there are 11 pints so u wanna multiply 16 and 11 to 176 ounces..

also i got it right on edge :P

Answer: 176

Step-by-step explanation:

You borrow $350 from your aunt and agree to repay her $400 ($350 principal + $50 interest) in 18 months. What interest rate (using simple interest, and to the nearest tenth) are you paying?

Answers

Answer:

The interest rate is 0.79% per month.

Step-by-step explanation:

Present value of borrowed amount = $350

Future value of borrowed amount = $400

Interest amount = $50

Time duration = 18 months or 1.5 years

Now we have to find the rate of interest by using the above information. Here below is the calculation of interest rate.

Interest rate =  P×R×T

50 = 350 × R×18

R = 0.007936

Or, R = 0.79% per month.

I REALLY NEED HELP PLEASE HELP ME

Answers

Answer:

52^8

Step-by-step explanation:

26^8 = 208827064576 + 26

if f(x)=4-3x*2 then f(-2) is

Answers

Answer:

16

Step-by-step explanation:

f(-2)=4-3(-2)*2

f(-2)=4-3(-4)

f(-2)=4+12

f(-2)=16

Simplify the expression: z8 * z-3

Answers

Answer:

z^5

Step-by-step explanation:

z^8  * z^-3

Since we are multiplying exponents and the bases are the same, we can add the exponents

z^(8-3)

z^5

What is the solution to this equation?
X-9 = 17
O A. x = 28
O B. x = 12
O c. x = 8
O D. x = 26

Answers

Answer:

Step-by-step explanation:

x -  9 = 17

x = 26

the answer is D

One year, Mr James travels 8 x 10^3 miles for his job. The next year he traveled 1 x 10^4 miles. How many more miles did he travel the second year than he did the first year? Please give me the answer and explain ToT

Answers

Answer:

2 000 miles

Step-by-step explanation:

In the first year, Mr James travels 8 x [tex]10^{3}[/tex] miles = 8 000 miles.

The second year, he traveled 1 x [tex]10^{4}[/tex] miles = 10 000 miles.

Comparing the distance covered in the two years,

10 000 miles - 8 000 miles = 2 000 miles

Therefore, Mr James travels 2 000 more miles in the second year than he did the first year.

Let's assume that Mr James conveys himself to his place of work all through the first year in his private car. But in the second year, there were some days in which the car was not available. So he had to go for his job by public transportation. This itch during those days, could cause an increase in the distance covered from his home to his place of work for the second year.

meaning of cube root

Answers

Answer:

The cube root of a number is a special value that, when used in a multiplication three times, gives that number. Example: 3 × 3 × 3 = 27, so the cube root of 27 is 3. See: Square Root. Cubes and Cube Roots.

The area of a sector is 30 m2 in a circle with radius 4 m. What is the arc length of the sector?

Answers

Answer:

15 m

Step-by-step explanation:

The area of the circle is

A =pi r^2

A = pi 4^2 = 16 pi

The area of the sector is 30

The fraction is

30/16 pi  

Take this time 2pi which are the  radians of a circle

30 /16 pi * 2 pi = 15/4

This is the number of radians the angle is

The arc length is s = r * theta where theta is in radians

s = r  theta

  = 4 * 15/4

  = 15

help please will mark brainliest

Answers

Answer:

i can't see that

Step-by-step explanation:

plzz sent me a clear photo

Answer:

1. 1

2. 1  ⋅  10 ^24

3. 1000000000

4. 1

5. 1  ⋅  10 ^20

6. 1  ⋅  10 ^30

7. 1  ⋅  10 ^27

8. 10000

9. 0.01

10. 0.0001

11. 0.1

Hopefully these are right, I simplified all of the problems...sorry if they are wrong

Step-by-step explanation:

Let n be a whole number, and consider the statements below. p: n is a multiple of two. q: n is an even number. Which of the following is equivalent to ~q → ~p? A. ~q → ~p B. q → p C. p → q D. ~p →~

Answers

Answer:

C. p -> q

Step-by-step explanation:

Just did this on Edge2020. Hope this helps :)


Need help on 3 thank you

Answers

Answer:

Cost: $10.00

Number of Tickets:26

Step-by-step explanation:

Each ticket is $0.50

20*$0.50=$10.00

For the tickets:

$13.00/$0.50=26 tickets

solve for x (2x+1) (x+29)​

Answers

The answer would be 2x^2+59x+29

Please solve for a ! 24+5a=-a+6(5a-4)

Answers

Answer:

a = 2

Step-by-step explanation:

First of all you should remove parentheses by using distribution

24 + 5a = - a + 30a - 24

Then you should collect all like terms

24 + 5a = 29a - 24

After you should move the terms so the variables are only on one side.

5a - 29a = -24 - 24

Collect like terms

-24a = -48

After dividing both sides you get

a = 2

This year, Zachary has been babysitting his young cousins after school for $70 a month. His uncle also
gave him an extra bonus of $100 for his excellent work. Since school started, Zachary has earned more
than $500. How many months ago did school start?

Answers

Answer:

8

Step-by-step explanation:

Which expression has the same value as -18-(-9)?
0 - 18+2
-12-(-3)
- 1945
O-8-(-4)

Answers

Answer:

the answer is -12-(-3)

Step-by-step explanation:

The expression equivalent to -18-(-9) is -12-(-3)

What are expressions?

An expression in maths is a sentence with a minimum of two numbers or variables and at least one maths operation.

Given is an expression, -18-(-9), we are given to find the expression equivalent to it,

-18-(-9) = -18+9 = -9

1) -18+2 = -16 (not equivalent)

2) -12-(-3) = -12+3 = -9

Therefore, we see, the value of expressions -12-(-3) and -18-(-9) are equivalent,

Hence, the expression equivalent to -18-(-9) is -12-(-3)

For more references on expressions, click;

https://brainly.com/question/14083225

#SPJ2

i need help its urgent its due today geometry

Answers

Rays, vertex, the vertex of the angle,BXC

What is the measurement of this angle

Answers

Answer:

Acute, 40 degree angle

Step-by-step explanation:

Take 90 - 50 and you get 40

I hope this helps you :)

-KeairaDickson

Answer:

40°

Step-by-step explanation:

Count how many 10s it takes to get from 90 degrees to 50 and you get your answer...

Christine wants to buy strawberries and raspberries to bring to a party. Strawberries cost $1.65 per pound and raspberries cost $2.25 per pound. She only has $15 to spend on berries. Which inequality represents the situation where she buys x pounds of strawberries and y pounds of raspberries?

Answers

Answer:

1.65x + 2.25y ≤ 15

Step-by-step explanation:

Strawberries = $1.65 per pound Raspberries = $2.25 per pound

Amount with Christine = $15

Let

pounds of strawberries = x

pounds of raspberries = y

PxQx + PyQy ≤ 15

1.65x + 2.25y ≤ 15

The inequality which represent the situation where she buys x pounds of strawberries and y pounds of raspberries is 1.65x + 2.25y ≤ 15

1.65x + 2.25y less than or equal to 15

Simple geometry equation

Answers

Answer:

106 or 4x+18  

Step-by-step explanation:

Since Ray QR is a bisector you can determine that PRQ and QRS are equal angles (def. of a bisector). Then, what I chose to do is graph them and find where they intercept which was 22. We now know 22 is x so we can substitute it into the equation and solve to deterine that our answer is 106.

22 = x, explanation:
x+ 31 = 3x-13
-x. -x
31=2x-13
+13.. +13
44-=2x
—. .—
2. 2
22= x

3/4 x - 1/2 - 4 = 12

Answers

Answer: x=22

Step-by-step explanation:

Step 1: Simplify both sides of the equation.

Step 2: Add 9/2 to both sides.

Step 3: Multiply both sides by 4/3.

27x + 45y and 7(9x + 7y) are these equivalent?

Answers

Answer:

No

Step-by-step explanation:

If you want to know whether they are equivalent or no, we will first have to find the value of both.

[tex]27x+45y[/tex]

Since there aren't any like terms here, your answer would be

[tex]27x+45y[/tex]

Let's solve for next one.

[tex]7(9x+7y)[/tex]

Let's add in parenthesis to 7.

[tex](7)(9x+7y)[/tex]

Now we will separate everything. And also you will have to add.

[tex](7)(9x)+(7)(7y)[/tex]

Now that we added we got our answer.

[tex]63x+49y[/tex]

Both values were not the same, so therefore, your answer is no, they are not equivalent.

Hope this helps!

No

Step-by-step explanation:

If you want to know whether they are equivalent or no, we will first have to find the value of both.

Since there aren't any like terms here, your answer would be

Let's solve for next one.

Let's add in parenthesis to 7.

Now we will separate everything. And also you will have to add.

Now that we added we got our answer.

Both values were not the same, so therefore, your answer is no, they are not equivalent.

Hope this helps!

Read more on Brainly.com - https://brainly.com/question/17379473#readmore

Jodi is considering taking online classes at two websites, LearnCenter and EduWorld. Each site requires that students pay a base membership fee, plus they charge a rate per class. The two graphs show the cost for taking online classes through the two websites. A graph titled Cost to Take Classes at LearnCenter has number of classes on the x-axis and cost in dollars on the y-axis. A line goes through points (0, 50) and (1, 70). A graph titled Cost to Take Classes at EduWorld has number of classes on the x-axis and cost in dollars on the y-axis. A line goes through points (0, 20) and (1, 60).

Answers

Answer:

B-LearnCenter charges a higher membership fee but a lower rate per class.

Step-by-step explanation:

Answer:B-learning for ever

Step-by-step explanation:

What is the smallest positive number that is prime and 10 less than a perfect square?

Answers

Answer:

The problem states that the answer cannot be a perfect square or have prime factors less than $50$. Therefore, the answer will be the product of at least two different primes greater than $50$. The two smallest primes greater than $50$ are $53$ and $59$. Multiplying these two primes, we obtain the number $3127$, which is also the smallest number on the list of answer choices. So we are done, and the answer is $\boxed{\textbf{(A)}\ 3127}$.

Step-by-step explanation:

Answer:

71

Step-by-step explanation:

I just did the AOPS question, you can see the attachment down below.

Hope this helped! :)

y=-4x - 8
6x + 2y=-10


I need the equation solved by substitution

Please show work :)

Answers

Answer:

x =1.28571428571

Step-by-step explanation:

6x + 2y = -10

y = -4x - 8

6x + 2(4x - 8) = -10

6x + 8x - 8 = -10

14x - 8 = 10

(we put together the like terms)

14x = 10 + 8

14x = 18

x =  [tex]\frac{18}{14}[/tex]

x = 1.28571428571

Other Questions
One-fourth of a number is five-eighths. Find the number. Draw a second resonance structure for the following ion (be sure to include the charges and all lone pairs)... + ..N=N=N .. .. 2. A laser printer prints 200 pages in 4minutes. How many pages does it print in 25minutes?1000800212501050 1.Convert as indicated.a) Change 246 days into hours.b) Change 2 years into hours.c) Change 1,463,200,000s into years. If there are 90 calories in Three-fourths cup of yogurt, how many calories are in 3 cups of yogurt? 30 calories 202 calories 270 calories 360 calories Dana skates for 20 seconds.how many laps will she skate? a 2 + 3 = 2 multi step equation part 5. Use the laws of sines. Please show work. Round to the nearest tenth. d=18+34 d=22t system of equations Why do the service women sprout mustaches? help plz and quickly What is the component on the motherboard that confirms all devices are in working order once the computer is turned on Explain why the concept "individual freedom" of liberal democracy is important using complete sentences. 7y = 21x + 14In the equation shown, what is the value of x when y 4?Give your answer as a fraction. please help me. 5(m-7)=-8(3m+8) 8x - 3(2x - 4) = 3(x - 6) When members of populations do not mate with each other or cannot produce fertile offspring, they are the last words of the Declaration of Independence say." we mutually pledge to each other our lives, our fortunes, and our sacred honor." what did this mean? Question 5 If you add me with another number our sums will be 13, our difference is 1, find us! Who is the Smaller Number?Blank 1: Question 6 If you divide me by 5 and decreace that value by 2, I am 25! What's My Number?Blank 1: Give two examples of the kinds of factors that can help to change a countrys population. Explain your answer in at least two to three sentences.